1) Maxillary Sinus Opens
a. Superior Meatus
b. Infundibulum
c. Inferior Meatus
d. None of the above
Answer : (b) Infundibulum
Reference: PL Dhingra 3rd Edition Page
2) Tensor of the Vocal cord
a. Cricothyroid
b. Posterior Crico arytenoids
c. Lateral crico arytenoids
d. Thyro arytenoids
Answer : (a) Cricothyroid
Reference: Gray 38th Edition Page 1645
*Action of Intrinsic Muscles of Larynx
- Varying the Rima Glottidis
- The posterior crico-arytenoids are the only
laryngeal muscles which open the glottis, rotating
the arytenoid cartilages laterally around an axis
passing through the crico-arytenoid joints, thus
separating the vocal processes and the attached
vocal folds. They also pull the arytenoids
backwards, assisting the cricothyroids to tense the
vocal folds. The most lateral fibres draw the
arytenoids laterally, so the rima glottidis becomes
triangular when the posterior crico-arytenoid
muscles contract.
- The lateral crico-arytenoids close the glottis by
rotating the arytenoids medially, to approximate
their vocal processes. The transverse arytenoid
pulls the arytenoid cartilages towards each other,
closing the posterior (intercartilaginous) part of the
rima glottidis.
- Regulating Tension in the Vocal Ligaments
- The cricothyroids stretch the vocal ligaments by
tilting the thyroid cartilage forwards and
downwards on the cricoid. Because the arytenoid
cartilages are anchored to the cricoid lamina, the
sagittally directed rotation of the thyroid cartilage
increases the distance between their vocal
processes and the anterior angle of the thyroid, so
lengthening the vocal ligaments. The cricoid is
usually held immovably against the vertebral
column by the cricopharyngeus during phonation
so that under these conditions it is the thyroid
cartilage which moves. During swallowing,
however, the cricopharyngeus relaxes, allowing the
cricoid to tilt forwards during laryngeal closure.
- The thyro-arytenoids draw the arytenoids towards
the thyroid cartilage, shortening and relaxing the
vocal ligaments. At the same time, they rotate the
arytenoids medially to approximate the vocal folds.
Their deeper fibres, the vocales, relax the posterior
parts of the vocal ligaments, their anterior parts
remaining tense and thus raising the vocal pitch.
For details of arytenoid movements consult Sellars.
- Modifying the Laryngeal Inlet
- The oblique arytenoids and aryepiglottic act as a
sphincter of the laryngeal inlet by adducting the
aryepiglottic folds and approximating the arytenoid
cartilages to the tubercle of the epiglottis.
- The thyro-epiglottic muscles widen the inlet by
their action on the aryepiglottic folds.
3) Prime modality of Rx of Naso Pharyngeal Ca –
a. Radiotherapy
b. Surgery
c. Chemotherapy
d. None of these
Answer : (a) Radiotherapy
Reference: PL Dhingra 3rd Edition Page 305 and
Bailey and Love 24th Edition Page 749, 750
4) Access to airway is gained during emergency by
a. Endotraceal tube
b. Emergency Tracheostomy
c. Crico thyroidotomy
d. All of these
Answer : (d) All of these
Reference: PL Dhingra 3rd Edition Page 386
5) Which of the following structure is seen in Oro
Pharynx
a. Pharyngotympanic tube
b. Fossa of Rosenmuller
c. Palatine Tonsil.
d. Piriform Fossa
Answer : (C ) Palatine Tonsil
Reference: Gray 3rd Edition Page 1729
6) Organ of corti is situated in
a. Basilar membrane
b. Utricle
c. Saccule
d. None of the above
Answer : (a) Basilar Membrane
Reference: Gray 38th Edition Page 1387
7) The presenting symptoms in majority cases of
Acoustic Neuroma is
a. Hearing Loss
b. Vertigo
c. Signs of Space occupying lesions
d. None of the above
Answer : (a) Hearing Loss
Reference: PL Dhingra 3rd Edition Page 144
8) The opening in case of Dacrocystorhinostomy is
made in
a. Superior Meatus
b. Middle Meatus
c. Inferior Meatus
d. None of the above
Answer : (b) Middle Meatus
Reference: Basak 3rd Edition Page 302
9) The most common diagnosis in a young boy with
history of profuse nasal bleeding is
a. Juvenile Naso pharyngeal angiofibroma
b. Nasal polyp
c. Deviated nasal septum
d. Cirrhosis
Answer : a) Juvenile Nasopharyngeal angiofibroma
Reference: Dhingra 3rd Edition Page 299
10) Left sided vocal cord palsy is commonly due to
a. Left hilar bronchial carcinoma
b. Mitral Stenosis
c. Thyroid Malignancy
d. Thyroid Surgey
Answer : a) Left hilar bronchial carcinoma
Reference: Dhingra 3rd Edition Page 360.
11) Why is a vocal cord pale ?
a. Vocal cord is muscle, lack of blood vessels
network
b. Absence of mucosa, no blood vessels
c. absence of sub mucosa, no blood vessels
d. absence of mucosa with blood vessels
Answer : c) absence of sub mucosa, no blood
vessels
Reference: Gray 38th Edition Chapter on larynx
12) Presbycusis is
a. Age associated vision loss
b. Age associated hearing loss
c. Both
d. None
Answer : b) Age associated hearing loss
Reference: Dhingra 3rd Edition Page 47
13) Acoustic dip in audiogram in noise induced
hearing loss is at
a. 4 KHz
b. 3 KHz
c. 2 KHz
d. 1 KHz
Answer : a) 4 KHz
Reference: Dhingra 3rd Edition Page 46
14) False positive fistula test
a. Perilymph fistula
b. Malignant sclerosis
c. Congenital syphilis
d. Cholesteatoma
Answer : c) Congenital Syphilis
Reference: Dhingra 3rd Edition Page 53
15) Reaction time for tymphanic reflex is
a. 20 to 30 ms
b. 30 to 40 ms
c. 40 to 160 ms
d. 600 to 800 ms
Answer : c) 40 to 160 ms
Reference: ganong 22nd Edition Page 179
16) Columella effect is seen in
a. Tympanoplasty
b. Septoplasty
c. Tracheostomy
d. None of the above
Answer : a) Tympanoplasty
Reference: Dhingra 3rd Edition Page 40
17) All are true regarding Reinke’s oedema except?
A. Usually caused by vocal abuse
B. There is collection of oedema fluid in the
subepithelial space
C. There is asymmetrical swelling of vocal
cords
D. Vocal cord stripping is the treatment
Correct answer : C. There is asymmetrical
swelling of vocal cords
Reinke’s oedema – diffuse symmetrical swelling of both
vocal cords
Reinke’s oedema is caused by vocal
abuse or smoking.
There is diffuse symmetrical swelling of both
vocal cords.
Vocal cord stripping is the treatment of choice.
Enough mucosa should be preserved for
epithelialization.
432) A 62-year-old man presents with painless, progressive
enlargement in the left side of his neck over the past 3 months.
He denies any fever, night sweats or chills. He denies any pain,
dysphagia, cough, hemoptysis, chestpain, shortness of breath,
history of trauma or dental infection. His past medical history is
significant for 100 pack-year history of smoking and daily
alcohol abuse. On examination, he is afebrile. He has a large 5
cm , hard, non-tender and matted lymphadenopathy on the left
lateral aspect of his neck . He has no other lymph node
enlargement. No rash. Reminder of the exam including oral
cavity examination is normal. There are no obvious tongue,
pharyngeal or tonsillar lesions. HIV and VDRL are negative. A
CT scan of the chest, abdomen and pelvis does not reveal any
lymphadenopathy or obvious mass lesions. Which of the
following is the most appropriate next step in management?
A) Open biopsy of the neck mass
B) Pan-endoscopy
C) Fine Needle Aspiration Cytology ( FNAC)
D) Test for HPV ( Human Papilloma Virus) and EBV ( Ebstein
Barr Virus)
E) Prescribe antibiotic therapy and re-evaluate in 4 weeks
Rate this:
28) A 32-year-old male boxer presents to the emergency room
after sustaining an injury during a local boxing match. He was
punched by the opponent over his right ear about six hours ago
. He complains of pain and swelling over his right ear. He has
no hearing deficit or tinnitus or headaches or blurred vision. He
denies nausea or vomiting. Past medical history is
unremarkable. On examination, his vitals are stable. His right
ear is red, warm and swollen with a medium sized anterior
auricular swelling which is tender to palpation. There is no facial
swelling. If untreated, which of the following would be most
likely course of this injury?
A) Spontaneous Resolution
B) Hearing Deficit
C) Permanent ear disfigurement
D) Otitis Externa
E) Mastoiditis
427) A 32-year-old male boxer presents to the emergency
room after sustaining an injury during a local boxing match. He
was punched by the opponent over his right ear about six hours
ago . He complains of pain and swelling over his right ear. He
has no hearing deficit or tinnitus or headaches or blurred vision.
He denies nausea or vomiting. Past medical history is
unremarkable. On examination, his vitals are stable. His right
ear is red, warm and swollen with a medium sized anterior
auricular swelling which is tender to palpation. There is no facial
swelling. Which of the following is the most appropriate
management?
A) Compressive dressing
B) Needle aspiration of the Hematoma
C) Cold compresses, analgesics and antibiotics
D) Incision and Drainage
E) Observe and await spontaneous hematoma resolution
325 ) A 87 year old Caucasian man is evaluated in your office
for progressive hearing loss. He reports defective hearing on
both sides that started more than a year ago and has gradually
worsened. The problem is particularly worse when he is in a
crowded or noisy environment. He also reports frequent
occurrence of roaring sounds and sounds similar to “ringing
bells” in his ears. On otoscopic examination, he has mild
cerumen impaction in bilateral ears. The Tympanic membranes
are intact. Which of the following is the most characteristic
feature seen with this condition?
A) Inability to hear “Vowel” sounds in the speech
B) Audiogram showing bilateral high-frequency sensory-neural
hearing loss
C) Weber Test showing lateralization towards Right ear
D) A negative Rinne test
E) Otosclerosis
357. Intermaxillary suture are sometimes raised
forming a
longitudinal midline ridge known as: (AFMC
2003)
a. Torus auditory
b. Torus mandibularis
c. Torus palatine
d. Torus maxillaris
358. Peritonsillar abscess is also known as:
(Karnataka [snip])
a. Retropharyngeal abscess
b. Tonsillar abscess
c. Quinsy
d. Thornwaldt’s abscess
359. Third molar caries with extension of the
lesion towards tonsillar
fossa and shift of tonsil reveals which of the
following
complication? (UP 2002)
a. Parapharyngeal abscess
b. Retropharyngeal abscess
c. Tonsillar abscess
d. Dental abscess
360. Peritonsillar abscess can extend
posteriorly into: (Delhi 2000)
a. Anterior triangle of neck
b. Parapharyngeal space
c. Posterior triangle on neck
d. Submaxillary space
361. Swelling between tonsillar area and
superior constrictor muscle
is know as: (AI 1991)
a. Quinsy
b. Dental abscess
c. Parapharyngeal abscess
d. Retropharyngeal abscess
362. The most common malignancy of the
oropharynx: (MP 2006)
a. Tonsil
b. Soft palate
c. Tongue base
d. Valleculae
363. Killiance dehiscence is seen in (JIPMER
1999)
a. Oropharynx
b. Nasopharynx
c. Cricopharynx
d. Vocal cords
364. Odynophagia is (MH PGM CET 2000)
a. Pain during swallowing
b. Difficulty in swallowing
c. Bad odour from mouth
d. Psychiatric disease
365. Openings of the tube of bronchoscope are
known as: (CMC 2002)
a. Holes
b. Apertures
c. Vents
d. Any of the above
366. Indications for tracheostomy are all
EXCEPT:
a. Acute epiglottitis
b. Maxillofacial trauma
c. Laryngeal malignancy
d. Extensive consolidation of lung
367. Most common complication of Tracheostomy
is: (JIPMER 2000)
a. Stenosis
b. Infection
c. Pneumonia
d. Respiratory failure
368. Collar stud abscess is seen in: (Orissa
2004)
a. Pyogenic cervical abscess
b. Peritonsillar abscess
c. Retropharyngeal abscess
d. TB lymphadenitis
369. A tracheostomised patient, with portex
tracheostomy tube, in
the ward, developed sudden complete blockage of
the tube.
Which of the following is best next step in the
management?
(AIIMS May 2004)
a. Immediate removal of the tracheostomy tube
b. Suction of tube with sodium bicarbonate
c. Suction of tube with saline
d. Jet ventilation
370. After a long-standing tracheostomy patient
developed almost
complete stenosis of trachea, treatment is:
(AIIMS 2001)
a. Tracheal dilation
b. Laser with stent
c. Surgery
d. Removal of stenosed part with anastomosis
371. Not true regarding pterygopalatine fossa:
(Orissa 2005)
a. Inferomedial to the foramen rotundum lies
the Vidian (Pterygoid)
canal, which connects the foramen lacerum to
the pterygopalatine
fossa.
b. Medially, the pterygopalatine fossa
communicates through the
pterygomaxillary fissure with the infratemporal
fossa
c. It communicates with the middle cranial
cavity through the foramen
rotundum and pterygoid canal.
d. It contains the sphenopalatine ganglion and
the third segment of
the maxillary artery.
372. White oral lesions are seen in EXCEPT:
(Kar 2002)
a. Leukoplakia
b. Keratosis
c. Addison’s disease
d. Candidiasis
373. All of the following cause a grey-white
membrane in the throat
EXCEPT (SGPGI 2005)
a. Streptococcal tonsillitis
b. Diphtheria
c. Ludwig’s angina
d. Adenoviral pharyngitis
374. True statement about faucial diphtheria
is: (Delhi 2001)
a. Pearly white membrane
b. Bleeding occurs if the membrane is tried to
remove
c. Bleeds when membrane is removed
d. Membrane can be removed easily
375. Collar stud abscess is seen in: (Orissa
2004)
a. Pyogenic cervical abscess
b. Peritonsillar abscess
c. Retropharyngeal abscess
d. TB lymphadenitis
376. Steeple sign is seen in: (SGPGI 2005)
a. Croup
b. Acute epiglottitis
c. Laryngomalacia
d. Quinsy
377. The antibiotic of choice in acute
epiglottitis pending culture
sensitivity report is: (Kar 2001)
a. Erythromycin
b. Rolitetracycline
c. Doxycycline
d. Ampicillin
378. Which of the following statement is true
for Ludwig’s angina?
(Delhi 2000)
a. It is an ischemic, painful condition of
Pectoralis minor muscle
b. It is diffuse cellulitis affecting the floor
of the mouth
c. Glycerin nitrate, local application is quite
helpful
d. None of the above
379. Submandibular space infection is known as:
(Manipal 2002)
a. Ludwig’s angina
b. Vincent’s angina
c. Parapharyngeal abscess
d. None
380. Which of the following structures are
preserved in radical neck
dissection? (Kar 2002)
a. Vagus nerve
b. Accessory Nerve
c. Internal jugular vein
d. Sternocleidomastoid muscle
381. Palatal myoclonus is seen in: (SGPGI 2005)
a. Epilepsy
b. Multiple sclerosis
c. Cerebellar infarction
d. Guillain Barre syndrome
382. What is the correct sequence of the
following while resuscitating
an infant with Foreign Body Airway Obstruction?
(UPSC 2004)
1. Chest thrust
2. Tongue-jaw lift
3. Back blows
Select the correct sequence form the codes
given below:
Codes:
a. 1,3,2
b. 3,2,1
c. 3,1,2
d. 2,1,3
383. A patient presented with a 3.5 cms size
lymph node enlargement,
which was hard and presented in submandibular
region.
Examination of the head and neck did not yield
any lesion. The
next investigation to be done: (CMC 2005)
a. CXR
b. Triple endoscopy
c. Supravital staining of oral mucosa
d. Laryngoscopy
384. Most common tumor to produce metastasis to
cervical lymph
nodes: (AIIMS MAY 2002)
a. Glottic Carcinoma
b. Nasopharyngeal carcinoma
c. Carcinoma Base of tongue
d. Carcinoma lip
385. Best diagnostic modality for
Nasopharyngeal angiofibroma is:
(AIIMS 1997)
a. Angiography
b. Biopsy
c. CECT
d. MRI
386. Investigation of choice for Nasopharyngeal
angiofibroma is:
(MP 2002)
a. MRI
b. CECT
c. Helical CT
d. Angiography
387. Causes of posterior soft tissue
nasopharyngeal mass include all
EXCEPT: (AMU 1996)
a. Plasmacytoma
b. Choanal atresia
c. Thornwaldt’s Cyst
d. Aneurysm of the carotid artery
e. Chordoma
388. All muscles of Palate are supplied by
cranial accessory nerve
except:
a. Tensor palati
b. Palatoglossus
c. Palatopharyngeus
d. None
389. All muscles of Pharynx are supplied by
cranial accessory nerve
except:
a. Thyropharyngeus
b. Inferior constrictor
c. Stylopharyngeus
d. Superior constrictor
390. All muscles of Tongue are supplied by
hypoglossal nerve EXCEPT:
a. Myoglossus
b. Palatoglossus
c. Genioglossus
d. Hyoglossus
391. Cranial accessory nerve supplies: (Orissa
99)
a. Palate
b. Pharynx
c. Palatoglossus
d. All
392. Mandibular nerve supplies: (PGI 89)
a. Tensor palati
b. Tensor tympani
c. Both
d. None
393. Most frequent site of branchial cyst is
at: (MH-SS-CET 2005)
a. Upper third of posterior border of
sternocleidomastoid
b. Lower third of anterior border of
sternocleidomastoid
c. Upper third of anteromedial border of
sternocleidomastoid
d. Supraclavicular fossa
394. The main differential diagnosis is second
branchial cleft cyst
include all EXCEPT:
a. Thyroglossal duct cyst
b. Cystic hygroma
c. Laryngocele
d. Ranula
395. Which of the following is not the site for
PARAGANGLIOMA? (AIIMS
Nov 2003)
a. Carotid bifurcation
b. Jugular foramen
c. Promontory in middle ear
d. Geniculate ganglion
396. Bano Begum presented with bleeding from
ear, pain, tinnitus and
increasing deafness. Examination revealed red
swelling/mass
behind the intact tympanic membrane that
blanches on pressure
with pneumatic speculum. Treatments for her
include all EXCEPT:
(AIIMS 2001)
a. Preoperative embolisation
b. Radiotherapy
c. Surgery
d. Interferons
397. Which of the following is the most
beneficial technique of using
chemotherapy with a course of radiotherapy in
head and neck
malignancies? (AIIMS NOV 2004)
a. Neo adjuvant chemotherapy
b. Adjuvant chemotherapy
c. Concurrent chemotherapy
d. Alternating chemotherapy and radiotherapy
398. Which implant is used in Nasopharyngeal
carcinoma:
(Kerala 2005)
a. Caesium
b. I-131
c. Gold
d. Iridium
399. A male aged 60 years has foul breath, he
regurgitates food that
is eaten 3 days ago. Likely diagnosis is:
(AIPGE 2001)
a. Zenker’s diverticulum
b. Meckel’s diverticulum
c. Scleroderma
d. Achalasia cardia
400. Main problem associated with carotid body
tumor operation is:
(Maharashtra 2000)
a. The tumor blends with bifurcation of carotid
artery
b. The tumor blends with jugular vein
c. Recurrence
d. Vaso vagal Shock
401. True about Carotid body tumor is all
EXCEPT: (AIIMS Nov 2004)
a. It is highly vascular mass
b. True cut biopsy is investigation of choice
c. Operative intervention is best avoided in
elderly patients.
d. Radiotherapy has no effect
402. Investigation of choice for carotid body
tumor? (AFMC 2004)
a. Angiography
b. USG with colour Doppler
c. CECT
d. MRI
403. A 40 year old patient is suffering from
carotid body tumor. Which
of the following is the best choice of
treatment for him?
(AIIMS 2004 may)
a. Excision of tumor
b. Radiotherapy
c. Chemotherapy
d. Carotid artery ligation both proximal and
distal to the tumor
404. Lower esophageal sphincter: (AIIMS 2005
may)
a. Has no tonic activity
b. Has a tone which is provided by the
sympathetic system
c. Relaxes on increasing abdominal pressure
d. Relaxes ahead of the peristaltic wave
405. A young patient presents with history of
dysphagia more to liquids
than solids. The first investigation you will
do is: (AIIMS may 2003)
a. Barium swallow
b. Esophagoscopy
c. Laryngoscopy
d. CT chest
406. Barium study in Trendelenburg position is
done for the diagnosis
of: (Kerala 2003)
a. Upper GIT disorders
b. Hiatal disorders
c. Esophageal disorder
d. Crohn’s disease
407. On a chest radiograph, a double shadow
behind the heart, signs
of aspiration pneumonia and absence of air in
stomach is
suggestive of: (KAR 2003)
a. Aneurysm of thoracic aorta
b. Achalasia
c. Carcinoma esophagus
d. Peptic ulcer (stomach)
408. “Rat-tail” appearance on barium swallow
examination is seen
in: (Maharashtra 2002)
a. Achalasia cardia
b. Carcinoma esophagus
c. Hiatus hernia
d. Diffuse esophageal spasm
409. “Cobble stone” esophagus is feature of:
(Orissa 94)
a. Candidial oesophagitis
b. Cytomegalovirus oesophagitis
c. Herpes oesophagitis
d. Hiatus hernia
410. “Rat tail Appearance” in barium swallow is
a feature of:
(KAR 2005)
a. Carcinoma of Esophagus
b. Achalasia Cardia
c. Barret’s esophagus
d. Chronic Reflux oesophagitis
411. ‘Cork-screw’ esophagus is seen in: (AI
2002; KAR 2001)
a. Vigorous achalasia cardia
b. Scleroderma
c. Diffuse esophageal spasm
d. Carcinoma esophagus
412. Dysphagia lusoria due to: (AIIMS Nov 2003;
Manipal 2002)
a. Esophageal diverticulum
b. Aneurysm of aorta
c. Esophageal web
d. Compression of esophagus by aberrant blood
vessel
413. Dysphagia lusoria is commonly due to:
(Manipal 2002)
a. Abnormal origin of left subclavian artery
b. Abnormal origin of right subclavian artery
c. Compression by aortic arch
d. Obstruction by foreign body
414. Corkscrew esophagus is seen in which of
the following
conditions? (AIPGE 2002)
a. Carcinoma esophagus
b. Scleroderma
c. Achalasia cardia
d. Diffuse esophageal spasm
415. Treatment for achalasia associated with
high rate of recurrence:
(AIPGE 2002)
a. Pneumatic dilatation
b. Laparoscopic myotomy
c. Open surgical myotomy
d. Botulinum toxin
416. Barrett’s oesophagus is: (AIPGE 2002)
a. Lower oesophagus lined by columnar
epithelium
b. Upper oesophagus lined by columnar
epithelium
c. Lower esophagus lined by ciliated epithelium
d. Lower esophagus lined by pseudostratified
epithelium
417. Adenocarcinoma of esophagus develops in:
(AIPGE 2002)
a. Barrett’s esophagus
b. Long standing achalasia
c. Corrosive structure
d. Alcohol abuse
418. Most common site for squamous cell
Carcinoma esophagus is:
(AIPGE 2001)
a. Upper third
b. Middle third
c. Lower third
d. Gastro esophageal junction
419. Dysphagia lusoria is due to: (NOV. 2003
AIIMS )
a. Oesophageal diverticulum
b. Aneurysm of aorta
c. Oesophageal web
d. Compression by aberrant blood vessels
420. A 30 years old male, presents with
symptoms of regurgitation of
foul smelling food, and dysphagia. He is
diagnosed to have
cricopharyngeal diverticulum. Management of
choice is:
( AIIMS 2001)
a. Excision of the diverticula
b. Cricopharyngeal myotomy alone
c. Myotomy with excision of the sac
d. Marsupialization of the sac
421. Bhanwarilal presents with feature of foul
smelling food
regurgitation, dysphagia and pain. He was
diagnosed to have
cricopharyngeal diverticulum. Treatment of
choice will be:
(AIIMS 2000)
a. LASER vaporization
b. Excision of the diverticulum with
cricopharyngomyotomy
c. Excision of the diverticulum
d. Marsupialisation of the diverticulum
422. Best substitute for esophagus after
esophagectomy is:
(MP 99; Ai 96)
a. Stomach
b. Jejunum
c. Right colon
d. Left colon
423. A young patients with dysphagia move for
liquids than solids. He
regurgitates food often at night. Radiography
shows a rat-tailed
appearance. Likely diagnosis is: (AIIMS May
1996)
a. Achalasia cardia
b. CA oesophagus
c. Zenker’s diverticulum
d. Diffuse esophageal spasm
424. A male aged 60 years has foul breath; He
regurgitates food that
is eaten 3 days ago a gurgling sound is often
heard on swallowing:
Likely diagnosis is: (AI 2001)
a. Zenkers diverticulum
b. Meckels diverticulum
c. Scleroderma
d. Achalasia cardia
425. All are true regarding Plummer Vinson
syndrome, except:
(AIIMS Dec 97)
a. Oesophageal web
b. Predisposes to malignancy
c. Koilonychia
d. Common in elderly males
426. The following are predisposing factors for
Esophageal carcinoma
except: (AI 1996)
a. Plummer-Vinson syndrome
b. Tylosis palmaris
c. Chronic Achalasia
d. Benzene therapy
427. Hyperkeratosis of palm and sole is seen
in: (AIIMS Dec 97)
a. Carcinoma colon
b. Hepatoma
c. Adenocarcinoma lung
d. CA oesophagus
428. The adenocarcinoma of esophagus-developed
in: (AI 2002)
a. Barrett’s esophagus
b. Long standing achalasia
c. Corrosive structure
d. Alcohol abuse
429. Adenocarcinoma of esophagus is commonly
found in: (AI 1998)
a. Achalasia acardia
b. Barrett’s oesophagus
c. Plummer Vinson syndrome
d. Chronic smoking
430. Most common site for squamous cell
carcinoma esophagus is:
(AI 2001)
a. Upper third
b. Middle third
c. Lower third
d. Gastro-esophageal junction
431. Which neo-adjuvant chemotherapy is used in
Esophageal
carcinoma: (AI 1996)
a. Cisplatin
b. Cyclophosphamide
c. Doxorubicin
d. Methotrexate
432. The commonest side effect of cisplatinum
in a patient using it for
esophageal carcinoma is: (AIIMS May 01)
a. Acute tubular necrosis
b. Thrombocytopenia
c. Hepatic failure
d. Cardiomyopathy
433. Constrictions normally present in
esophagus are all EXCEPT:
(MH-PGM-CET-2000):
a. 10 cm from the incisor teeth
b. 15 cm from the incisor teeth
c. 25 cm from the incisor teeth
d. 40 cm from the incisor teeth
434. Which of the following play vital role in
preventing gastro
oesophagus reflux? (PGI 2001)
a. Sphincteric action of lower esophageal
muscle fibers or fibers of
right cura.
b. Mucosal flap produced by muscularis mucosa
of the stomach.
c. Difference between intrathoracic and
intraabdominal pressure.
d. Acute gastroesophageal angle.
435. The most common complication seen in
hiatus hernia is: (AI 2005)
a. Oesophagitis
b. Aspiration pneumonitis
c. Volvulus
d. Oesophageal stricture
436. The hypopharynx includes all the following
EXCEPT: (JIPMER 2003)
a. Pyriform fossa
b. Epiglottis
c. Post cricoid region
d. Valeculae
357. Ans: (c) (Torus palatine)
358. Ans: (c) (Quinsy)
359. Ans: (a) (Parapharyngeal abscess)
360. Ans: (b) (Parapharyngeal space)
361. Ans: (a) (Quinsy)
362. Ans: (a) (Tonsil)
363. Ans: (Cricopharynx)
364. Ans: (a) (Pain during swallowing)
365. Ans: (c) (Vents)
366. Ans: (d) (Extensive consolidation of lung)
367. Ans: (b) (Infection)
368. Ans: (d) (TB lymphadenitis)
369. Ans: (a) (Immediate removal of the
tracheostomy tube)
370. Ans: (d) (Removal of stenosed part with
anastomosis)
371. Ans: (b) (Medially, the pterygopalatine
fossa communicates
through the pterygomaxillary fissure with the
infratemporal
fossa)
372. Ans: (c) (Addison’s disease)
373. Ans: (c) (Ludwig’s angina)
374. Ans: (b) (Bleeding occurs if the membrane
is tried to remove)
375. Ans: (d) (TB lymphadenitis)
376. Ans: (a) (Croup)
377. Ans: (a) (Erythromycin)
378. Ans: (b) (It is diffuse cellulitis
affecting the floor of the mouth)
379. Ans: (a) (Ludwig’s angina)
380. Ans: (a) (Vagus nerve)
381. Ans: (b) (Multiple sclerosis)
382. Ans: (c) (3,1,2)
383. Ans: (b) (Triple endoscopy)
384. Ans: (b) (Nasopharyngeal carcinoma)
385. Ans: (c) (CECT)
386. Ans: (b) (CECT)
387. Ans: (b) (Choanal atresia)
388. Ans: (a) (Tensor palati)
389. Ans: (c) (Stylopharyngeus)
390. Ans: (b) (Palatoglossus)
391. Ans: (d) (All)
392. Ans: (a), (b)
393. Ans: (c) (Upper third of anteromedial
border of sternocleidomastoid
394. Ans: (d) (Ranula)
395. Ans: (d) (Geniculate ganglion)
396. Ans: (d) (Interferons)
397. Ans: (a) (Neo adjuvant chemotherapy)
398. Ans: (d) (Iridium)
399. Ans: (a) (Zenker’s diverticulum)
400. Ans: (a) (The tumor blends with
bifurcation of carotid artery)
401. Ans: (b) (True cut biopsy is investigation
of choice)
402. Ans: (c) (CECT)
403. Ans: (a) (Excision of tumor)
404. Ans: (d) (Relaxes ahead of the peristaltic
wave)
405. Ans: (a) Barium Swallow
406. Ans: (b) (Hiatal disorders)
407. Ans: (b) (Achalasia)
408. Ans: (b) (Carcinoma esophagus)
409. Ans: (a) (Candidial oesophagitis)
410. Ans: (a) (Carcinoma of Esophagus)
411. Ans: (c) (Diffuse esophageal spasm)
412. Ans: (d) (Compression of esophagus by
aberrant blood vessel)
413. Ans: (a) (Abnormal origin of left
subclavian artery)
414. Ans: (d) (Diffuse esophageal spasm)
415. Ans: (b) (Laparoscopic myotomy)
416. Ans: (a) (Lower oesophagus lined by
columnar epithelium)
417. Ans: (a) (Barrett’s esophagus)
418. Ans: (b) (Middle third)
419. Ans: (d) (Compression by aberrant blood
vessels)
420. Ans: (c) (Myotomy with excision of the
sac)
421. Ans: (b) (Excision of the diverticulum
with cricopharyngomyotomy)
422. Ans: (a) (Stomach)
423. Ans: (a) (Achlasia cardia)
424. Ans: (a) (Zenker’s diverticulum)
425. Ans: (d) (Common in elderly male)
426. Ans: (d) (Benzene therapy)
427. Ans: (d)(Ca Esophagus)
428. Ans: (a) (Barrett’s Esophagus)
429. Ans: (b) (Barrett’s esophagus)
430. Ans: (b)(Middle third)
431. Ans: (a) (Cisplatin)
432. Ans: (a) (Acute tubular necrosis)
433. Ans: (a) 10 cm
434. Ans: (a), (b), (c), (d)
435. Ans: (a) (Oesophagitis)
436. Ans: (d) Valeculae
1.Merciful anosmia is seen in
a.Atrophic rhinitis
b.Allergic rhinitis
c.Ethmoidal polyposis
d.Wegener’s granulomatosis
Explanation: Atrophic rhinitis(Ozaena) is a chronic
inflammation of nose characterized by atrophy of
nasal mucosa and turbinate bones. The nasal
cavities are roomy and full of foul smelling crusts.
Disease is commonly seen in females and starts
around puberty. There is foul smell from the nose
making the patient a social outcast though patient
himself is unaware of the smell due to marked
anosmia (merciful anosmia) which accompanies
these degenerative changes.
2.Acoustic neuroma most commonly arises from
a.Nervous intermedius
b.Superior vestibular nerve
c.Inferior vestibular nerve
d.Cochlear nerve
Explanation: Time and Again, i have seen many
students fighting over what could be the answer for
this particular question. The answer was superior
vestibular nerve before a few years and now it is
Inferior vestibular nerve based on the recent
research done. That particular paper which was
presented by 2 doctors from Tokyo medical and
dental university can befound here.
3.The surgical procedure currently recommended
for ethmoidal polyposis is
a.Functional endoscopic sinus surgery
b.Caldwel-Lucs procedure
c.Sinoscopy
d.Antral lavage
4.Which surgery is commonly performed for unsafe
CSOM?
a.Myringotomy
b.Myringoplasty
c.Modified radical mastoidectomy
d.None of the above
5.Frisch bacillus causes
a.Rhinoscleroma
b.Rhinosporidiosis
c.Rhinophyma
d.Lupus vulgaris
Explanation:
Rhinoscleroma, or simply Scleroma, is a chronic
granulomatous bacterial disease of the nose that
can sometimes infect the upper respiratory tract.It
most commonly affects the nasal cavity—the nose is
involved in 95-100 per cent of cases—however, it
can also affect the nasopharynx, larynx, trachea,
and bronchi. Slightly more females than males are
affected and patients are usually 10 to 30 years of
age. Rhinoscleroma is considered a tropical disease
and is mostly endemic to Africa and Central
America, less common in the United States. It is
caused by Klebsiella rhinoscleromatis—subspecies
of Klebsiella pneumoniae— a gram-negative,
encapsulated, nonmotile, rod-shaped bacillus
(diplobacillus), member of the Enterobacteriaceae
family. It is sometimes referred to as the "Frisch
bacillus," named for Anton von Frisch who
identified the organism in 1882. It is contracted
directly by droplets or by contamination of material
that is subsequently inhaled.
6.Emergency tracheostomy is not indicated in
a.Acute severe asthma
b.Foreign body in respiratory tract
c.Laryngeal cancer producing stridor
d.Bilateral vocal cord paralysis
7.Labyrinthine artery is a branch of
a.anterior inferior cerebellar artery
b.posterior inferior cerebellar artery
c.middle meningeal artery
d.basilar artery
Explanation:The labyrinthine artery (auditory
artery, internal auditory artery), a long slender
branch of the anterior inferior cerebellar artery
(85%-100% cases) or basilar artery (15% cases),
arises from near the middle of the artery; it
accompanies the vestibulocochlear nerve through
the internal acoustic meatus, and is distributed to
the internal ear.
8.Cork screw esophagus seen on barium swallow is
a feature of
a.achalasia cardia
b.diffuse esophageal spasm
c.nut-cracker esophagus
d.gastroesophageal reflux
9.The most frequent congenital laryngeal lesion is
a.laryngomalacia
b.subglottic hemangioma
c.congenital subglottic stenosis
d.laryngeal web
10.Multiple nasal polypi in children should guide
the clinician to search for underlying
a.Mucoviscidosis
b.Celiac disease
c.Hirschsprung disease
d.Sturge Weber syndrome
Cystic fibrosis (also known as CF or
mucoviscidosis) is an autosomal genetic disease
affecting most critically the lungs, and also the
pancreas, liver, and intestine. It is characterized by
abnormal transport of chloride and sodium across
epithelium, leading to thick, viscous secretions.....
Mucus in the paranasal sinuses is equally thick and
may also cause blockage of the sinus passages,
leading to infection. This may cause facial pain,
fever, nasal drainage, and headaches. Individuals
with CF may develop overgrowth of the nasal tissue
(nasal polyps) due to inflammation from chronic
sinus infections. Recurrent sinonasal polyps
can occur in as many as 10% to 25% of CF
patients. These polyps can block the nasal
passages and increase breathing
difficulties...............
. A 21-year-old swimmer with an unremarkable medical
history comes to the clinic because of pain and itching in
his left ear. He also reports some moist discharge from
this ear. The symptoms began two days ago and have
worsened. He denies any hearing loss, tinnitus, or vertigo.
Pain is elicited when the left auricle is pulled superiorly
and when the left tragus is pressed inward. The left ear
canal is edematous and erythematous, and the
considerable yellowish debris present in the canal
obstructs visualization of the tympanic membrane.
What is the most appropriate next step in the management
of this situation?
A) Irrigate ear canal with hydrogen peroxide
B) Clean ear canal with a cerumen wire loop or cotton
swab
C) Apply topical antibiotics to ear canal with dropper
D) Prescribe analgesic medication
E) Refer to otolaryngologist for stent placement
The correct answer is: B
Explanation:
External otitis arises secondary to skin maceration
and failure of the skin-cerumen barrier that provides
natural protection against infection. The condition is
associated with swimming, excessive cleaning or itching of
the ear canal, and usage of objects that occlude the ear
canal (e.g., hearing aid, earphones). The organisms most
commonly responsible for external otitis are those found in
normal skin flora, including P. aeruginosa and S. aureus.
Diagnosing external otitis is done clinically, based upon
the history and physical examination. Pain caused by
tragal pressure or superior movement of the auricle is
considered a classic finding of external otitis. Treatment of
the condition must always first begin with careful cleaning
of the ear canal, as healing is significantly aided by
removal of the cerumen, desquamated skin, and purulent
debris. Therefore it would be best to clean the ear canal
with a cerumen wire loop or cotton swab.
(Choice A) Irrigation of the ear canal with diluted
hydrogen peroxide would also be a good choice, but only
if the tympanic membrane is visible and intact.
(Choice C) Topical antibiotics such as polymyxin or
ciprofloxacin are frequently used and have a good effect in
patients with external otitis; however, the ear canal should
be cleaned first so that the antibiotics will have maximal
impact.
(Choice D) Analgesic medication may be needed to
relieve the pain that accompanies external otitis. NSAIDs
are usually sufficient, although narcotics may be
necessary in some cases. However, this is not the first
step taken in managing the situation.
(Choice E) Referral to an otolaryngologist for stent
placement is indicated in those patients who have
advanced or severe disease. The stent allows for topical
medications to reach deeper into the swollen ear canal.
Educational Objective:
External otitis is associated with swimming, excessive
cleaning or itching of the ear, or usage of occlusive
devices. Treatment must first begin with cleaning of the
ear canal with a cerumen wire loop or cotton swab.
Irrigation with hydrogen peroxide is an acceptable
alternative cleaning method if the tympanic membrane is
visualized and intact.
Wanna try more, then continue solving these wonderful
set of Mcqs.
2.An 82-year-old Caucasian female comes to the
emergency department for the evaluation of right ear pain
and drainage for the past two days. She saw her primary
care physician approximately one week ago for decreased
hearing from the right ear, and he performed aural
irrigation to remove impacted cerumen in her right ear.
She has a history of hypertension, diabetes mellitus, and
rheumatoid arthritis. She denies any history of smoking or
alcohol abuse. Her medications include
hydrochlorothiazide, enalapril, glyburide, and low-dose
prednisone. On physical examination, her temperature is
37.2C (99F), blood pressure is 146/74 mmHg, heart rate is
100/min, and respiratory rate is 16/min. There is marked
tenderness with motion of the right earlobe. Purulent
discharge and granulation tissue is noted on the floor of
the right external auditory canal at the osseocartilaginous
junction. The tympanic membrane appears intact.
(I)Which of the following is the most likely cause of the
above findings?
A) Acute otitis media
B) Malignant otitis externa
C) Ramsay Hunt syndrome
D) Acute mastoiditis
The correct answer is: B
Explanation:
This is a classic presentation of malignant otitis
externa (also known as malignant external otitis), which is
an invasive infection of the external auditory canal and the
bones forming the skull base. The infection usually begins
as external otitis and progresses rapidly to involve the
adjacent bones at the base of the skull. Patients have
marked pain (otalgia) and purulent drainage or discharge
from the ear (otorrhea). The finding of granulation tissue at
the floor of the bone-cartilage junction in the external
auditory canal is pathognomonic of this condition. The
tympanic membrane is usually intact.
Malignant otitis externa is typically seen in elderly
patients with diabetes mellitus. Patients with HIV disease
and other immunocompromised states are also at a higher
risk of having malignant otitis externa. A number of
patients with malignant otitis externa usually have an
associated history of aural irrigation for the removal of
cerumen.Pseudomonas aeruginosa is the usual causative
organism in these patients. An untreated infection can
progress rapidly to involve the skull base,
temporomandibular joint, and cranial nerves, causing
osteomyelitis and cranial nerve palsies.
(Choice A) Acute otitis media is associated with an
inflamed, erythematous, bulging and immobile tympanic
membrane due to the presence of fluid in the middle ear.
(Choice C) Ramsay Hunt syndrome (also known as
herpes zoster oticus) is an ear manifestation of a
reactivated varicella zoster virus. It is characterized by a
triad of ear pain, vesicles in the external auditory canal,
and ipsilateral facial paralysis.
(Choice D) Acute mastoiditis is usually seen as a
complication of acute otitis media.
Educational Objective:
Malignant otitis externa is typically seen in elderly, diabetic
patients, and is characterized by severe pain and the
presence of granulation tissue on the floor of the external
auditory canal at the osseocartilaginous junction.
(II)Which of the following is the most appropriate treatment
for this patient’s condition?
A) Topical tobramycin
B) Oral amoxicillin
C) Intravenous acyclovir
D) Intravenous ciprofloxacin
E) Acetic acid drops
The correct answer is: D
Explanation:
Pseudomonas aeruginosa is the usual causative
organism in almost all the cases of malignant otitis
externa. Anti-pseudomonal antibiotic therapy is therefore
the treatment of choice. Fluoroquinolones (ciprofloxacin),
anti-pseudomonal penicillins (piperacillin, ticarcillin) with or
without aminoglycosides, and third generation
cephalosporins (ceftazidime) are all effective in the
treatment of malignant external otitis. All patients should
be treated with intravenous antibiotics initially, and then
switched to oral antibiotics (depending on the clinical
response), to complete 6-8 weeks of antibiotic therapy.
(Choice A) Topical antibiotics have no role in the
treatment of malignant external otitis.
(Choice B) Oral amoxicillin is not effective
against Pseudomonas aeruginosa, and should not be
used.
(Choice C) Intravenous acyclovir is used in the
management of patients with Ramsay Hunt syndrome.
(Choice E) Topical acetic acid (and other acidifying
agents) is generally used in patients with mild external
otitis. It has no role in the treatment of malignant otitis
externa.
Educational Objective:
Systemic anti-pseudomonal antibiotics should be used in
the treatment of malignant external otitis.
3.A 54-year-old Caucasian chef presents to clinic
complaining about "a new problem with really bad
dizziness." She says that when she first sat up in bed this
morning, she suddenly felt very unstable and as if her
body was spinning in space. She was overcome by
nausea but did not vomit, and then the episode passed
after approximately one minute. She had similar episode
when she was working in the restaurant kitchen. She has
not had an episode like this before. She has not been ill
recently or in contact with any ill people to her knowledge.
Her past medical history is significant for mild chronic
obstructive pulmonary disease and ulcerative colitis.
Current medications include albuterol, prednisone,
mesalamine, and ibuprofen. She has a remote thirty pack-
year history of cigarette smoking and drinks alcohol only
on social occasions. Her temperature is 36.8C (98F),
blood pressure is 124/72 mm Hg (sitting) and 120/68 mm
Hg (standing), pulse is 75/min, and respirations are
14/min. Positional nystagmus is noted on physical
examination.
Which of the following measures is the most appropriate
course of action?
A) Order CT scan of head
B) Order MRI scan of head
C) Perform canalith repositioning procedure
D) Prescribe promethazine
E) Refer for plugging of the posterior canal
The correct answer is: C
Explanation:
Benign paroxysmal positional vertigo (BPPV) is
defined as an abnormal feeling of motion triggered by
certain provocative positions. The condition is most often
attributed to canalithiasis, or the presence of calcium
"rocks" within the posterior semicircular canal. If there is
substantial debris in the canal, then linear accelerations
(eg, gravity) may cause the endolymph to move
inappropriately. This results in the inaccurate sensation of
spinning subsequent to movement of the head. Although
the condition is often idiopathic, it can also be associated
with head trauma or vestibular pathologies. BPPV should
first be treated with the canalith repositioning procedure
(Epley's maneuvre)(Choice C), which is a series of
maneuvers that move particles out of the posterior
semicircular canal and into the utricle.
BPPV is a clinical diagnosis. Further testing is not
indicated with typical posterior canal BPPV presentation.
Imaging of the head (Choices A and B) is indicated if a
mass lesion, hemorrhage, stroke, or Meniere’s disease is
thought responsible for the patient’s vertigo.
Promethazine (Choice D) is a nonselective
antihistamine used in the treatment of nausea or vomiting.
It may be of help in patients with vertigo but addresses
only this particular symptom and not the cause.
Plugging of the posterior canal (Choice E) is a
surgical procedure used in patients with intractable
symptoms of BPPV. In most cases, the procedure reduces
the functionality of the posterior canal without affecting
hearing.
Educational Objective:
Benign paroxysmal positional vertigo should first be
treated with the canalith repositioning procedure, which is
a series of maneuvers that moves particles out of the
posterior semicircular canal and into the utricle.
1. Which are the three most common organisms causing
acute sinusitis ?
2. What are Ducts of Rivinus ?
3. Wharton’s duct?
4. What is the most probable diagnosis when the
findings are as follows : erythematous aryepiglottic folds,
grey granulation tissue in the interarytenoid region and
posterior thirds of vocal cords, ulcers in the posterior thirds
of vocal cords ?
5. What is the deformity seen in post mandibulectomy
patients called as ?
6. Which anti-microbial drug should not be given in
Infectious mononucleosis ?
7. What is your first diagnosis is an elderly male who
comes with progressively worsening stridor of 3 months
duration ?
8. A 3 yr old boy comes with the complaint of foul
smelling sero sanguinous discharge from one nostril. What
is your first diagnosis ?
9. In which condition is steeple sign seen ?
10. Expand Gd-DTPA.
11. What is ELSA ?
12. What is Dorello's canal ?
13. What is the part of the tuning fork that we place on the
mastoid called ?
14. Which is the causative organism of acute epiglottitis ?
15. Which ear drops will you prescribe for a person with
right ear TM rupture due to trauma ?
16. What is the most common cause for intraoperative
bleeding in adenoidectomy ?
17. Which is the most feared complication during removal
of a foreign body from the nose of a child ?
18. What is the procedure of choice for severe air hunger
in supine position in a man with supraglottic growth ?
19. "cobblestone" esophagus seen in?
20. What is the lateral rhinotomy incision also known as ?
21. What are the temperatures of water used in the
Bithermal test for vestibular function ?
22. Romberg's test positive signifies _______ disorder or
_________ disorder.
23. What is chondrodermatitis chronicus pinna also
known as ?
24. Collar stud abscess seen in?
25. Cork screw esophagus seen in?
26. What is Woolnerian tip?
27. The Cody tack operation is used in the treatment of?
28. Wullstein’s classification?
29. Blainville ears?
30. Submandibular space infection is known as?
ANSWERS
1. Pneumococcus, H.influenzae, Moraxella catarrhalis
2. Minor ducts of the sublingual salivary gland .Some
directly open into oral cavity& some unite to form major
duct of Bartholin.
3. Submandibular gland duct
4. Laryngeal findings in pachyderma laryngis.
5. Andy Gump deformity .This anatomic defect results
from resection of the anterior mandibular arch without
adequate reconstruction.
6. Ampicillin (can cause rash)
7. Laryngeal Cancer.
8. Foreign body Nose
9. Acute laryngotracheobronchitis .(check this post for all
other signs in ENT)
10. Gadolinium Diethylene Triamine Pentaacetic Acid (It
is a type of contrast agent.A substance used in magnetic
resonance imaging (MRI) to help make clear pictures of the
brain, spine, heart, soft tissue of joints, and inside bones. )
11. Endoscopic Ligation of Sphenopalatine
Artery .Endoscopic ligation of the sphenopalatine artery
(ESPL) has recently become the treatment of choice for
refractory epistaxis.
12. Abducent nerve canal .
13. Footpiece (not base)
14. Hemophilus influenzae type B
15. Don't give any ear drops
16. Adenoid tags
17. Aspiration into airway
18. Cricothyrotomy
19. Moniliasis.In the diagnosis of Candida esophagitis,
double contrast esophagography shows a sensitivity of
about 90% , demonstrating discrete plaque-like filling
defects which have a finely nodular and granular,
distinctive cobblestone or snakeskin-like appearance and
correspond to the distinctive white plaques seen at
endoscopy. These
plaques consist of heaped-up areas of necrotic epithelial
debris or actual colonies of C. albicans on the esophageal
mucosa; the esophagus per se has an irregular or shaggy
appearance .
20. Moure incision. The incision is started from the inner
extremity of the eyebrow, descending along the lateral wall
of the nose over the naso labial fold. It is curved up to the
alar margin. The classic Moure's incision should not
extend into the vestibule of the nose. The advantage of
this incision is that it can be extended above and below to
facilitate better exposure of midface, anterior skull base
and orbit. The incision heals with minimal scarring.
21. 30 C and 44 C
22. Vestibular or Posterior column
23. Also known as Winkler's disease . Chondrodermatitis
nodularis chronicis helicis is an painful, inflammatory
nodule of the external ear. nonwhites have been noted
occasionally to have lesions in areas other than the helix,
such as the antihelix or antitragus. The lesions are believed
by several researchers to relate to trauma or sun damage.
The nodules are more commonly reported on the right ear,
which is believed to be the preferred resting side during
sleep.
24. TB lymphadenitis.Cervical lymphadenopathy is also
termed “scrofula”, meaning “glandular swelling” in Latin.
The nodes coalesce, break down and perforate the deep
fascia, resulting in the characteristic collar-stud abscess,
which this case resembles.
25. Diffuse esophageal spasm.
26. Darwin's tubercle is a congenital ear condition which
often presents as a thickening on the helix at the junction of
the upper and middle thirds.However Darwin himself
named it the Woolnerian tip, after Thomas Woolner, a
British sculptor who had depicted it in one of his sculptures
and had first theorised that it was an atavistic feature.
27. Meniere’s disease
28. Classification is
Type 1 ossicular chain intact, only ear drum is
repaired.Myringoplasty is synonymous
with tympanoplasty type 1
Type 2 Graft placed on incus or remnant of malleus
Type 3 INCUS & MALLEUS absent & the grafted
drum is placed in contact with the HEAD of
STAPES ( COLUMELLA EFFECT)
Type 4 refers to the baffle effect of a tympanic
membrane protecting the round window,
while leaving open the mobile stapes
footplate of the oval window
Type 5 fenestration of the lateral semicircular
membrane in the presence of a fixed footplate
and an intact tympanic membrane.
29. Asymmetry in size or shape of the auricles.
30. Ludwig’s angina
3 comments:
Email ThisBlogThis!Share to TwitterShare to
FacebookShare to Pinterest
Labels: ENT, MCQs
Tuesday, March 3, 2009
Fistula test, Hennebert sign &
Tullio Phenomenon .
Fistula test or Perilymphatic fistula test
: The fistula test is performed
by applying positive and negative pressure to the intact
eardrum using a pneumatic otoscope or by pressing
tragus.
Used to detect Perilymph fistula.
Positive Result(indicates
Negative Result(Normal)
Perilymphatic Fistula)
when positive pressure is when positive pressure is No changes noted.
applied with the applied with the pneumatic
pneumatic otoscope otoscope
+
Onset of Nystagmus towards
ipsilateral ear.
when negative pressure is when positive pressure is No changes noted.
applied with the applied with the pneumatic
pneumatic otoscope otoscope
+
Nystagmus also reverses &
changes its direction towards
contralateral ear.
The type of nystagmus seen can be deducted from the
picture below.
Ok first let me try to explain you the figure given above.
First lets take a normal person as an example ,in him
when,
This will be the direction of slow phase of
SCC which is stimulated
nystagmus in ear with fistula
right horizontal semicircular canal is it causes movement of both eyes towards the
stimulated opposite(left) side.
right posterior semicircular canal is it causes downward movement of both eyes
stimulated as well as intorsion of right eye& extorsion
of left eye.
right anterior semicircular canal is stimulated it causes upward movement of both eyes as
well as intorsion of right eye& extorsion of
left eye.
Now lets come to left vestibular system
left horizontal semicircular canal is it causes movement of both eyes towards the
stimulated opposite(right) side.
left posterior semicircular canal is stimulated it causes downward movement of both eyes
as well as intorsion of left eye& extorsion of
right eye.
left anterior semicircular canal is stimulated it causes upward movement of both eyes as
well as intorsion of left eye& extorsion of
right eye.
So, if there is fistula in right ear,then
if fistula on right horizontal SCC(that means when positive pressure applied via pneumatic
when positive pressure is applied, the right otoscope causes slow component of
horizontal SCC will get stimulated) nystagmus towards opposite ear .
As per convention direction of nystagmus is
towards fast component, therefore we say
that nystagmus is towards ipsilateral ear.
if fistula on right posterior SCC when positive pressure is applied to right ear,
then according to the diagram we see that it
will cause nystagmus with slow component
towards downside& intorsion of eyeball.
As per convention direction of nystagmus is
towards fast component, therefore we say
that the nystagmus is upbeating& extorion
type.
Perilymphatic fistula:
A perilymph fistula (PLF) is an abnormal opening between
the air-filled middle ear and the fluid-filled inner ear. It may
occur due to a defect in one of three locations:
Oval Window –(most common site)
Stapedectomy surgery (for otosclerosis)
Head trauma or barotrauma (pressure injury)
Acoustic trauma
Round window -
Barotrauma -- SCUBA diving, airplane
pressurization
Congenital malformations (such as Mondini
dysplasia)
Otic capsule--
Another possible location for a fistula is in the bone of the
ear (the otic capsule). This is a rare condition where the
bone between the ear and brain area is missing or thin,
causing symptoms very similar to that of a round or oval
window fistula. Problems in the otic capsule that may
cause a perilymph fistula include:
Superior canal dehiscence syndrome (anterior
SCC)
Cholesteatoma
Fenestration -Another type of bony fistula can
occur after a surgical procedure called fenestration previously
done for otosclerosis;
Temporal bone fracture
Micro-fissure
The most common type of otic capsule fistula is located
just above the superior semicircular canal and is called the
superior canal dehiscence syndrome.
False positive fistula test(Hennebert
False negative fistula test
sign)
Congenital syphilis In Dead ear ( inner ear is damaged), there
(here stapes footplate is hypermobile, so even will be NO response even if a Perilymphatic
small pressure changes in ear, cause fistula exists.
excessive movement of stapes footplate &
excessive stimulation of utricular macule)
25% cases of Meneire’s disease. Also seen when cholesteatoma covers the
(here in 25% cases of meniere’s ,fibrous site of fistula & doesn’t allow pressure
bands form connecting to utricular macule to changes to be transmits to labyrinth.
stapes footplate)
HENNEBERT'S SIGN:
It is a false positive fistula test i.e, when there is no
evidence of middle ear disease causing fistula of horizontal
semicircular canal.
It is seen in 25% cases of meniere's disease or
congenital syphilis.
Hennebert sign- pressure induced nystagmus &
Hennebert symptom- pressure induced dizziness.
Tullio’s phenomenon:
Sound-induced vestibular symptoms such as vertigo,
nystagmus, oscillopsia, and postural imbalance .
Tullio's phenomenon is seen mainly in:
Superior canal dehiscence,
Meniere's syndrome,
vestibulofibrosis.
other causes of perilymph fistula,
post fenestration surgery(for otosclerosis).
14 comments:
Email ThisBlogThis!Share to TwitterShare to
FacebookShare to Pinterest
Labels: ENT
Friday, February 27, 2009
Onodi cell & Haller cell
Onodi cell(sphenoethomoid cell):
This is formed by lateral and posterior
pneumatization of the most posterior ethmoid cells over the
sphenoid sinus.
Because the Onodi cells are posterior ethmoid cells
that are positioned superolateral to the sphenoid sinus
,the optic nerve & carotid artery may often course through the
lateral aspect of onodi cell instead of sphenoid sinus proper.
Kainz and Stammberger defined an Onodi cell as a
posterior ethmoid cell with an endoscopically visible bulge of
the optic canal.
The vulnerability of the optic nerve with or without the
presence of an Onodi cell is further compounded by the thin
lamina papyracea in the posterior ethmoid area .
This anatomic variation is found in 8-14% of
cases according to studies using CT in association with recent
developments in endoscopic sinus surgery.
The presence of Onodi cells increases the chance
that the optic nerve and/or carotid artery would be exposed (or
nearly exposed) in the pneumatized cell.
The optic nerve, and more rarely, the internal carotid
artery, may be exposed within or lie immediately adjacent to
such an air cell.
During endoscopic sinus surgery attempts to
localise the sphenoidal sinus via instrumentation through
the posterior most ethmoidal air cells can lead to optic
nerve, and even, internal carotid artery, injury.
Haller cell (infraorbital cell or infraorbital extension of ethmoid cell):
The Haller cell is usually situated below the orbit in
the roof of the maxillary sinus.(i.e., in the orbital floor).
It occurs in 10% of people.
It is a pneumatized ethmoid cell that projects along
the medial roof of the maxillary sinus.
As it is closely related to ethmoidal infundibulum
enlarged Haller cells may contribute to narrowing of the
ethmoidal infundibulum and recurrent sinus disease.
16 comments:
Email ThisBlogThis!Share to TwitterShare to
FacebookShare to Pinterest
Labels: ENT
Thursday, February 12, 2009
Eponymous Signs in ENT
Below are the compilation of all the eponymous signs in
ENT from all over the net.Read & enjoy..
AQUINO'S SIGN is the blanching of the tympanic mass
with gentle pressure on the carotid artery.Seen in Glomus
tumors .
BATTLE SIGN- Bruising behind ear at mastoid region,
due to petrous temporal bone fracture (middle fossa #)
BEZOLD'S SIGN / SYMPTOM
Inflammatory edema at the tip of the mastoid process in
mastoiditis
BOCCA’S SIGN - Absence of post cricoid crackle(Muir’s
crackle) in Ca post cricoid
BROWNE'S SIGN
Refers to the blanching noted when applying positive
pressure{with Siege's speculum} to the tympanic
membrane of a patient with Glomus tumor.
BRYCE SIGN - If combined laryngocele & external
laryngocele is presenting as a neck mass, compression
will cause a hissing sound as the air escapes from it into
the larynx. This test is fraught with danger in cases of
combined laryngoceles because air from the external
component may get forced into the internal component
causing acute airway obstruction.
DELTA SIGN
Lateral sinus thrombosis on CT or MRI with contrast
shows an empty triangle appearance of the thrombosed
sinus surrounded by contrast enhanced dura{since
contrast may flow around the clot to outline the periphery
of the sinus}. It is also called as empty triangle
sign.
DODD’S SIGN/CRESCENT SIGN- X-ray finding-Crescent
of air between the mass and posterior pharyngeal
wall. positive in AC ployp Negative in Angiofibroma
FURSTENBERG'S SIGN-Positive in
Encephaloceles.Owing to the intracranial connection,
there is pulsation and expansion of the mass with crying,
straining, or compression of the jugular vein
(Furstenberg test).This is used to differentiate Nasal
Encephaloceles from other congenital midline nasal
masses like Nasal Gliomas.
GRIESINGER'S SIGN-Erythema and oedema posterior to
the mastoid process resulting from septic thrombosis of
the mastoid emissary vein. seen in lateral sinus
thrombosis
HALO SIGN/ HANDKERCHIEF SIGN - A finding in CSF
rhinorrhea when CSF is mixed with Blood.
In patients with head trauma, a mixture of blood and
CSF may make the diagnosis difficult.
CSF separates from blood when it is placed on filter
paper, and it produces a clinically detectable sign: the ring sign,
double-ring sign, or halo sign.
CSF will separate from blood when the mixture is
placed on filter paper resulting in a central area of blood with an
outer ring or halo.
Blood alone does not produce a ring.
The best ring is obtained with a 50: 50 mix of blood
and CSF.
More importantly, they found that the presence of a
ring was not exclusive for CSF.
Blood mixed with tap water, saline, and rhinorrhea
fluid also produced a ring.
The halo sign does occur, but clearly does not clinch
the diagnosis.
HITSELBERGER’S SIGN - In Acoustic neuroma- loss of
sensation in the postero-superior part of external auditory
meatus supplied by Arnold’s nerve( branch of Vagus nerve
to ear )
HOLMAN MILLER SIGN, ANTRAL SIGN-
The anterior bowing of the posterior wall of the antrum
seen on lateral skull film .Pathognomic for juvenile
nasopharyngeal angiofibroma.
HONDOUSA SIGN–X-ray finding in Angiofibroma.
indicating infratemporal fossa involvement
characterised by widening of gap between ramus of
mandible and maxillary body.
HENNEBERT'S SIGN
It is a false positive fistula test when there is no
evidence of middle ear disease causing fistula of
horizontal semicircular canal. It is seen in 25% cases of
meniere's disease or congenital syphilis.In 25% cases of
Meneire’s ,fibrous bands form connecting utricular macule
to stapes footplate. In syphilis due to hypermobile stapes
footplate.[Hennebert sign- pressure induced
nystagmus, Hennebert symptom- pressure induced
dizziness]
IRWIN MOORE’S SIGN——– positive squeeze test
in chronic tonsillitis.
LAUGIER'S SIGN-Blood behind the eardrum
suggests basilar skull fracture.
LEUDET'S SIGN-Inflammation of the eustachian tube can
produce a bright clicking sound heard by the examiner
through the otoscope while the patient experiences it as
tinnitus.caused by reflex spasm of the tensor palati
muscle.
LIGHT HOUSE SIGN—A small pin hole perforation with a
pulsatile ear discharge is seen in Acute suppurative
otitis media.
LYRE’S SIGN - splaying of carotid vessels( at junction of
External & internal carotid artery) in carotid body tumor.
MILIAN’S EAR SIGN- Erysipelas can spread to
pinna(cuticular affection), where as cellulitis cannot.
Cellulitis and erysipelas manifest as areas of skin
erythema, edema and warmth in the absence of
underlying suppurative foci.
They differ in that erysipelas involves the upper dermis
and superficial lymphatics, whereas cellulitis involves the
deeper dermis and subcutaneous fat.
As a result, erysipelas has more distinctive anatomic
features than cellulitis; erysipelas lesions are raised above
the level of surrounding skin, and there is a clear line of
demarcation between involved and uninvolved tissue.
Classic descriptions of erysipelas note "butterfly"
involvement of the face.
Involvement of the ear (Milian's ear sign) is a
distinguishing feature for erysipelas since this region
does not contain deeper dermis tissue.
OMEGA SIGN -INFANTILE OMEGA SHAPED
EPIGLOTTIS SEEN IN LARINGOMALACIA.
Paul Dudley White's winking ear lobe sign-Movement
of the ear lobe coincident with the pulse
suggests tricuspid insufficiency.
PHELP’S SIGN - loss of crest of bone (as seen in CT-
scan) between carotid canal and jugular canal in glomus
jugulare.
RAT TAIL SIGN /“Bird-beak” sign-Sign in barium
swallow of achalasia. The oesophagus is dilated, and
contrast material passes slowly into the stomach as the
sphincter opens intermittently. The distal oesophagus has
a narrow segment and the image resembles a bird's beak.
This is in contrast to the rat's tail appearance of carcinoma
of oesophagus.Barium swallow shows characteristic rat
tail appearance with irregular mucosa margins in
carcinoma esophagus.
RISING SUN SIGN
There is red vascular hue seen behind the intact tympanic
membrane. it is seen in glomus tumour, high jugular
bulb and aberant carotid artery in the floor of middle
ear.
RACCOON SIGN-Indicate subgaleal hemorrhage,and not
necessarly base of skull .
SCHWARTZ SIGN
It is also called flamingo flush sign. it is seen because of
increased vascularity in submucous layer of promontory
in active phase of otosclerosis(otospongiosis).
STEEPLE SIGN- X-ray finding in Acute
laryngotracheobronchitis (CROUP).The steeple sign is
produced by the presence of edema in the trachea, which
results in elevation of the tracheal mucosa and loss of the
normal shouldering (lateral convexities) of the air
column.
STANKIEWICK’S SIGN - indicate orbital injury during
FESS. fat protrude in to nasal cavity on compression of
eye ball from outside .
TEAR DROP SIGN
Seen in Orbital floor fracture. It is defined as tear drop
shaped opacification seen hanging from the roof of the
maxillary sinus on water's view. The floor of the orbit is the
most common portion of the orbit to sustain fracture. A
classic radiographic finding in blow-out fractures is the
presence of a polypoid mass (the tear-drop) protruding
from the floor of the orbit into the maxillary antrum The
tear-drop represents the herniated orbital contents,
periorbital fat and inferior rectus muscle.
THUMB SIGN
It is a thumb like impression (due to enlarged epiglottis)
seen on X-ray lateral view neck in patients with
acute epiglottitis.Direct visualization of the epiglottis by
laryngoscope, if attempted, reveals a beefy red,
edematous epiglottis.
TRAGUS SIGN
In acute otitis externa there is marked tenderness when
tragus is pressed against the pinna.
TEA POT SIGN is seen in CSF rhinorrhoea.This could be
related to the relationship of the sphenoid ostium to the
sinus floor. The sphenoid ostium lies at an appreciable
distance anterosupe-rior from the sinus floor. An increase
in the CSF rhinorrhea therefore occurs in a case of
sphenoid sinus leak when the patient bends forward as an
increasing amount of CSF gains access to the
ostium "teapot" sign.
uvula pointing sign - uvula points to side of palatal palsy
Uvula pointing sign- seen in rhinoscleroma .when
scleroma involve nasopharynx ,uvula point towards roof of
nasopharynx.
WOODS SIGN—– palpable jugulodigastric lymphnodes.
80 comments:
Email ThisBlogThis!Share to TwitterShare to
FacebookShare to Pinterest
Labels: ENT
Bezold eponyms in ENT
1. Bezold's triad
1. Diminished perception of the deeper tones,
2. retarded bone conduction, and
3. negative Rinne's test,
pointing, in the absence of objective signs, to otosclerosis
2. Bezold's abscess
abscess deep in the neck parapharyngeal space
associated with suppuration in the mastoid cells
3. Bezold's sign / symptom
Inflammatory edema at the tip of the mastoid process in
mastoiditis
4. Bezold's ganglion
An aggregation of nerve cells in the interatrial septum
5. Bezold's mastoiditis
Mastoiditis with perforation medially into the digastric
groove and forming a deep neck abscess
1 comment:
Email ThisBlogThis!Share to TwitterShare to
FacebookShare to Pinterest
Labels: ENT
Wednesday, January 21, 2009
Delphian node
A midline prelaryngeal (cricotyhroid)
lymph node.
Receives lymphtic drainage from larynx & thyroid
gland.
Can be enlarged in thyroid carcinoma or advanced
larygeal cancer.
This node gets its name from Greek mythology. The
Oracle of Delphi, called Pythria, was a Priestess who would
answer questions posed to her, never concealing nor revealing
the truth. Thus, an enlarged Delphian node may, or may not,
indicate the presence of thyroid cancer.
No comments:
Email ThisBlogThis!Share to TwitterShare to
FacebookShare to Pinterest
Labels: ENT
Sunday, January 11, 2009
Lines of Dolan and the
elephants of Rogers
What are the lines of Dolan?
They are three anatomic contours best seen on the
Waters view (occipitomental view )of the face, and they were
first popularized by Dolan et al.
As you can see, the 3 lines of Dolan lead the eye
along some facially important structures.
Lee Rogers pointed out that the 2nd and 3rd lines
together form the profile of an elephant.
ZMC fractures = Zygomaticomaxillary complex
fractures or Tripod fractures
2 comments:
Email ThisBlogThis!Share to TwitterShare to
FacebookShare to Pinterest
Labels: ENT
Thursday, August 21, 2008
Assessment of Hearing
In this post we will deal with the techniques followed to
assess auditory function
we should be able to know theses 4 things after doing the
tests:
type of hearing loss:
degree of hearing loss
Degree of hearing loss hearing loss range(dB)
Not significant 0-25dB(adults)
0-15dB(children)
Mild 26-40dB
Moderate 41-55dB
Moderately severe 56-70dB
Severe 71-90dB
Profound more than 91dB
total
site of lesion
cause
Tests for hearing:
These 3 tests are non-specific & they don't indicate the
type of deafness.they only give a rough idea about the
hearing loss.
Finger friction test rubbing or snapping thumb &
finger close to patients ear
Watch test clicking watch brought near the
patients ear
Speech test patient stands with his test ear
towards examiner at distance of
6metres.
Tuning fork tests:remember these ,its very important
512Hz tuning fork is ideal(forks of lower frequency -
produce sense of bone vibration forks of higher frequency-
shorter decay time )
When we test air conduction ,we actually check the
functioning of both conductive & sensorineural(cochlea)
When we test Bone conduction ,we actually
measure only cochlear function
Normally hearing through air conduction is louder &
heard twice as long as through bone conduction route(AC
better than BC)
Rinne test:
method:To perform this test, a 512Hz vibrating tuning fork
is placed on the mastoid bone and then moved next to the
external ear. The patient indicates at which of the two sites
the sound is louder.
principle:Sound transmitted
through an external ear traverses
the middle ear and is perceived by
the cochlea (inner ear). Sound can
be transmitted directly to the
cochlea, skipping the external and middle ear, by placing
the vibrating tuning fork on the mastoid bone directly
behind the ear. This is the basis for the Rinne hearing test.
normal conductive SN deafness
deafness
Rinne AC>BC BC>AC AC>BC
(Rinne +) (Rinne -)
note:
rinne (-)ve 256Hz fork 512Hz fork 1024Hz fork
minimum 15dB 30dB 45dB
air-bone gap
False negative Rinne :
in Severe Unilateral sensorineural hearing loss.
Patient does not perceive sound by air conduction
,but responds to bone conduction
response to bone conduction is because the patient
perceives sound from opposite ear because of transcranial
transmission of sound.
corrected by :masking opposite ear with Barany's
noise box ,so that transcranial transmission of sound is not
perceived.
note:A Barany noise box can also be
used to see noise based vertigo (Tullio
phenomenon). This commercially
available box simply makes a loud (100
dB) noise. When the box is slowly
moved towards the patient's
symptomatic ear, the vertiginous symptoms may be re-
created.
Weber test:
method:Place the tuning fork in the center of the forehead
and the physician asks the patient where he or she hears
it.
principle:
The occlusion effect is
responsible for this phenomenon.
Sound conducted through bone causes
the cochlea, the ossicular chain, and
the air in the external auditory canal to
vibrate. Some lower frequencysound, as
produced by the 512 Hz tuning fork,
escapes from the canal. When the ear is
occluded, these frequencies cannot escape and the sound
seems to become louder.
it is the occlusion effect, rather than elimination of
environmental sound, that is responsible for the improved
bone conduction threshold when occluding a normal ear.
Middle ear effusion and ossicular chain disruptions
cause a "mass loaded" middle ear, with lowering of the inherent
resonant frequency. Ossicular chain fixation causes a phase
shift in the sound wave. Both cause preferential
transmission of lower frequencies to the cochlea
normal conductive SN deafness
deafness
Weber not lateralised lateralised to lateralised to
poorer ear better ear
note:lateralisation of sound in Weber test with a tuning fork
of 512Hz implies either
conductive loss of 15-20dB in ipsilateral ear (or)
sensorineural deafness in contralateral ear
Absolute bone conduction test(ABC) test:
method:
patients bone conduction compared to that of
examiner(presuming that examiner has normal hearing)by
keeping on mastoid.
External auditory meatus of both patient & examiner is
occluded by pressing tragus inwards ,this is to prevent
external ambient noise entering through air
conduction route.
normal conductive SN deafness
deafness
ABC test hear the fork hear the fork reduced
for same for same
duration as duration as
examiner examiner.
Schwabach test:
method:same as ABC test,but meatus is not occluded.
normal conductive SN deafness
deafness
Schwabach equal lengthened in reduced
patient(due
to absence of
external
ambient
noise the
patient hears
it for longer
time)
Bing test::
determine whether closing of ear canal results in occlusion
effect.
The Bing test can simulate unilateral (one-sided)
conductive hearing loss results by placing a finger in
one ear while performing the Weber test.
method:tuning fork placed on mastoid while examiner
alternately closes & opens ear canal by pressing tragus
inwards.
principle:same principle as Weber test(occlusion effect)
normal conductive SN deafness
deafness
Bing test louder(when no effect louder
occluded) (bing
negative)
Gelle's test:
method:A vibrating tuning fork is applied over the
mastoid process; if it is heard, the air in the external
auditory canal is compressed, by means of a Siegle's
speculum.
a test of the mobility of the ossicles.
principle:
when air pressure is increased in ear canal, it
pushes tympanic membrane inside & leading to increased
stiffness of the ossicular chain & thus decreases hearing.
And when pressure is again released ,it again
normalises.
normal stapes SN
fixation(otosclerosis) deafness
Gelle's test decreased in does not alter decreased in
intensity intensity
after after
increasing increasing
pressure pressure
Audiometric tests:
Pure tone audiometry:
used to measure the auditory threshold of an
individual
pure tone-a single frequency sound is used while
testing ,audiometer-an electronic device which produces pure
tones.
Audiometer: There are two types of audiometers
widely used. They are:
1. Those that require a subjective response on the
part of the patient and
2. Those that require no subjective response from the
patient.
Examples include:
1. Pure tone audiometer is the classic example of the
first type
2. Impedence audiometer / BERA (Brainstem Auditory
Evoked Responses audiometer) are examples of the
second type.
The frequencies generated are 125, 250, 500,750,
1000, 1500, 2000, 3000, 4000, 6000 and 8000 Hz.
Intensity is the level of sound power measured in
decibels; loudness is the perceptual correlate of intensity.
Frequency is cycles per unit of time. Pitch is the
perceptual correlate of frequency. Frequency is measured in
hertz, which are cycles per second.
Usually frequencies of 250-8000 Hz are used in
testing because this range represents most of the speech
spectrum, although the human ear can detect frequencies from
20-20,000 Hz.
The hearing level (HL) is quantified relative to
"normal" hearing in decibels (dB), with higher numbers of dB
indicating worse hearing. The dB score is not really percent
loss, but neverthless 100 dB hearing loss is nearly equivalent to
complete deafness for that particular frequency. A score of 0 is
normal. It is possible to have scores less than 0, which indicate
better than average hearing.
note:In a normal PTA audiogram we see that both
AC(air conduction) & BC(bone conduction) are at the same 0dB
level.But we know that AC is better than BC.So you may get a
doubt how come they are same here.Actually In a clinic a
calibrated audiometer is used to present the correct
intensity for each tone such that 'normal hearing' registers
as 0 dB HL (audiometric zero) .This is done for the ease of
reading the audiogram report.& standardisation
0 dB Hearing Level at 1000 Hz = 7 dB SPL
Pure tone air conduction testing:
This is a measurement of air
conduction thresholds of audibility.
Pure tone air conduction threshold is
tested using head phones:
method:
note:when establishing threshold ,2
choices are ascending & descending
way to change intensity.
ascending(Hughson - Westlake ascending
technique)- begins with stimuli that are below patient's
threshold & intensity is increased until patient
responds.
descending- stimuli presented first are above
patient's threshold & intensity is decreased until the
patient no longer responds.
but both have drawbacks=
in descending technique the patient might
continue to respond to stimuli ,when he no longer
perceives (false + response)
in ascending technique the patient may fail to
respond even when stimuli are audible.(false -
response)
Modified Hughson - Westlake technique :this procedure
uses an ascending technique to determine threshold
.but each threshold search is preceeded by a
descending familiarization trial.
ex:"Up 5-down 10" method of threshold estimation
1. The better ear is tested first in order to determine
the need for masking.
2. Start with a 1000 Hz tone at a level above the
threshold to allow easy identification of the tone. This tone
is selected because it is an important speech
frequency, and the patient is less apt to mistake the
frequency. To ensure the subject is familiar with the
task, present a tone of 1000 Hz that is clearly
audible (e.g. at 40 dB HL for a normally hearing subject or
approximately 30 dB above the estimated threshold for a
subject with a hearing impairment,)
3. If the patient is suspected to be having a profound
hearing loss then the testing should be started with 250Hz
frequency. This is because of the fact that the individuals
with profound hearing loss often have testable hearing only
in the low frequency range.
4. Next, test 2000, 4000, 8000, 500 and 250 Hz in that
order
5. As the threshold levels are being reached, a check
should be made for the existance of abnormal tone decay.
This is done by sustaining the tone for several seconds
longer than usual. If the index finger drops before the tone
is discontinued, abnormal tone decay should be suspected.
6. "Up 5-down 10" method =The starting intensity of
the test tone is reduced in 10 dB steps following each
positive response, until a hearing threshold level is reached
at which the subject fails to respond. Then, the tone is
raised by 5 dB, if the subject hears this increment, the tone
is reduced by 10 dB; if the tone is not heard then ti is raised
by another 5 dB increment. This 5 dB increment is always
used if the preceding tone is not heard, and a 10 dB
decrement is always used when the sound is heard. The
threshold is defined as the faintest tone that can be heard
50% or more of the time, and is established after several
threshold
crossings.
Testing of the second ear should begin with the last
frequency used to test the first ear. There is no need to start
again with a 1000 Hz tone because if one side of the heard has
learned the listening task, the other side knows it as well. The
test is terminated after all desired frequencies have been
examined.
Bone conduction audiometry:
This is an important measurement of hearing
threshold using a bone vibrator. This helps to
differentiate conductive from sensorineural hearing loss.
The equipment necessary is just a bone vibrator
connected to the audiometer. The bone vibrator is placed
over the mastoid process of the side to be tested. The
auditory threshold is assessed as described for air
conduction assessment. The only difference is that the
better hearing ear should be masked using a masking
tone delivered via a head phone.
Limitations of bone vibrators
1. Bone vibrators tend to emit more
sound than vibration at frequencies
above 2000 Hz If testing at 3000 and 4000
Hz it is preferable to insert an ear
plug (eg E.A.R. plug as used for hearing protection
purposes) into the test ear canal or cover the test ear with
a supra-aural earphone. This attenuates the air borne
radiation from the bone vibrator to a satisfactory
degree. Failure to occlude the ear canal at high test
frequencies is likely to lead to inaccurately acute bone
conduction thresholds, resulting in a false air-bone
gap in the audiometric results. The canal must not be
occluded at test frequencies below 3000 Hz since this may
produce the “occlusion effect” in which bone conduction
thresholds are improved.
2. Testing is not recommended at frequencies below
500 Hz because the subject’s threshold may relate to
hearing at the second or third harmonic rather than the
fundamental.
3. In bone conduction audiometry high frequencies
cannot be used for testing. Frequencies above 4000 Hz
cannot be used because they are beyond the vibrating
capabilities of the bone vibrator.
Masking
Masking presents a constant noise to the nontest
ear to prevent crossover from the test ear. The purpose of
masking is to prevent the nontest ear from detecting the signal
(line busy), so only the test ear can respond.
When a signal is presented to the test ear, the signal
may also travel through the head and reach the cochlea on the
other side. However the intensity of the signal from the test to
the nontest ear can be reduced by the mass of the head. This
signal reduction is called interaural attenuation.(‘transcranial
transmission loss’)
For bone conduction, the interaural attenuation
may be as low as 0 dB because the bones of the skull are
very efficient at transmitting sound. Thus, any suspected
difference in bone conduction between the test and nontest
ears requires masking. (ie,Masking is done in all Bone
conduction studies)
Crossover occurs when sound presented to the test
ear travels across the head to the nontest ear. This occurs at
approximately 40 dB for circumaural earphones across all
frequencies.Interaural attenuation for air conduction can range
between 40 and 80 dB. Masking should be used if the
difference in air conduction in one ear and bone
conduction in the other ear is 40 dB or greater.(ie,masking
required when there is difference b/w the 2 ears of minimum
40dB in air conduction threshold )
When the difference in the thresholds of the two ears
is greater than the transcranial transmission loss, cross-hearing
may occur and the apparent threshold of the worse ear is in fact
a ‘shadow’ of the better ear.
Narrow band masking noise is used
Audiograms:
red indicates right ear
blue indicates left ear.
eg: high-frequency sensorineural hearing loss in the
right ear.
Audiogram depicting a mild rising conductive hearing
loss in the left ear:
The 3 types of hearing loss can be differentiated as
follows:
Conductive hearing loss has normal bone-
conduction thresholds, but air-conduction thresholds are poorer
than normal by at least 10 dB.
Sensorineural hearing loss has bone- and air-
conduction thresholds within 10 dB of each other, and
thresholds are higher than 25 dB HL.
Mixed hearing loss has conductive and
sensorineural components.Pure-tone air-conduction thresholds
are poorer than bone-conduction thresholds by more than 10
dB, and bone-conduction thresholds are less than 25 dB
Speech audiometry:
test patient's ability to hear & understand speech is
measured.
different parameters are measured like:
(a)speech recognition threshold :Speech reception
threshold:or Spondee threshold
minimum intensity at which 50% of words repeated
correctly by patient.
Spondee words are used.Spondees are two
syllable words with equal accent (emphasis) on each
syllable. Baseball, northwest, oatmeal and hotdog are
examples of spondees.
The term speech recognition threshold is synonymous
with speech reception threshold. Speech recognition
threshold is the preferred term because it more accurately
describes the listener's task.
SRT should be within 10dB of PTA.(since SRT
encompasses far more frequencies than PTA , SRT is less
than PTA )
If the SRT is significantly better than the PTA, the
possibility of pseudohypoacusis(patient is not co-
operating properly) should be considered
1. note:Pure tone average(PTA)=average of pure tone
threshold of 3 separate frequencies (500 ,1000 ,2000Hz) as
measured by pure tone audiometry.
(b)Speech detection threshold:
Speech-awareness threshold (SAT) is also known
as speech-detection threshold (SDT).
The objective of this measurement is to obtain
the lowest level at which speech can be detected at least
half the time. This test does not have patients repeat words;
it requires patients to merely indicate when speech stimuli are
present.
Speech materials usually used to determine this
measurement are spondees
note:SRT differs from SDT in that SDT is merely the
level at which the individual becomes aware that
speech is present & SRT is level at which speech is
loud enough for the individual to understand it.
(c)speech recognition score :
Speech discrimination score or word recognition
score(when words used) or sentence recognition score(when
sentences used)
the lowest intensity speech stimulus that an individual can detect at
least 50% of the time.
discrimination means judging two things whether they are same or not ,but
this is not done in this test .therefore the term "speech discrimination score' not used
now.
here patient doesn't repeat words ,just a measure
of patient's ability to understand speech.
here Phonetically balanced words(PB)
used=single syllable words=pin,bus
phonetically balanced means that the distribution
of phonetic elements in list of words approximates the
distribution found in everyday conversations.
list of 50 words given & then the number of
correctly heard words multiplied by 2 to get score.
done at supra-threshold level i.e, at 30-40dB
above SRT.
in normal people & those with conductive
deafness have a high score of 90-100%.
90-100% within normal range
75-90% slight difficulty
60-75% moderate difficulty
50-60% poor discrimination
<50% very poor
PB max: performance intensity function for PB words.
here PB scores are taken against several levels of
speech intensity at 30-40dB above SRT(suprathreshold level) &
maximum score (PB max) is obtained & also note the intensity
at which it is obtained.
this is used clinically to set volume of hearing aid.
Rollover phenomenon:
defined as the decay of the
speech discrimination score (greater than 20% change) with
increased stimulus intensity,
When present, this finding is indicative of
a retrocochlear lesion
When the ear with a damaged nervous system is
stimulated with a very loud sound, the nerves may be unable to
handle the increased signal load. Consider this analogy. If you
injured your arm, you might be able to lift a small weight, but
would be unable to lift a heavy weight. Similarly, the damaged
VIII nerve may be able to transmit a conversational level
speech signal, but “tire out” when faced with the demand of
sending strong, sustained messages.
Impedence audiometry:::Acoustic
immittance.Immittance is a term derived from the terms
for two inversely related processes for assessing middle
ear function, impedance and admittance. Impedance is
the resistance to the flow of acoustic energy. Admittance
is the ease of which acoustic energy can flow. A middle
ear with low impedence (high admittance) more readily
accepts acoustic energy, whereas a middle ear with high
impedence (low admittance) tends to reflect energy
consists of -
1. tympanometry
2. acoustic reflex measurements
The primary purpose of impedence audiometry is to
determine the status of the tympanic membrane and the
middle ear.
The secondary purpose of this investigation is to
evaluate the acoustic reflex pathway which include the 7th
and 8th cranial nerves and the brain stem.
principle:
when a sound strikes tympanic membrane ,some of
the sound energy is absorbed while rest is reflected.A stiffer
membrane would reflect more sound energy than a compliant
(loose) one.
so by changing the pressure in a sealed external ear
canal & then measuring the reflected sound energy ,we
measure the compliance or stiffness of tympano-ossicular
system & thus find the healthy or diseased status of middle ear.
The maximum compliance occurs when the
pressure of the external auditory canal and the middle ear
becomes equal.
Only at this pressure maximal acoustic
transmission occur through the middle ear & minimum
sound energy is reflected back.
Tympanograms:
A tympanogram is a graphic representation of the
relationship of external auditory canal air pressure to
impedance
Pressure in the external auditory canal is varied from -
200 daPa(decaPascal=mmH2O) through +200daPa while
monitoring impedance
The Jerger system is the most commonly used
classification system for tympanograms
Type A. The peak compliance occurs at or near
atmospheric pressure indicating normal pressure in the middle
ear. There are three subgroups. Compliance peak is -150 to
+100 daPa
A - normal shape reflects a normal mechanism
.immittance is 0.2-2.5 millimhos(unit of conductance)
AD - A deep curve with a tall peak indicates an
abnormally compliant middle ear, as seen in ossicular
dislocation or erosion, or loss of elastic fibers in the
tympanic membrane. immittance is less than 0.2 mmhos
AS - A shallow curve indicates a stiff system, as
in otosclerosis. immittance is more than 2.5 mmhos
Type B - No sharp peak, with little or no variation in
impedance over a wide range, usually secondary to non-
compressible fluid in the middle ear (otitis media), tympanic
membrane perforation or obstructing cerumen.
This Type B curve must always be interpreted in
conjunction with the ear canal volume. Average ear canal
volume in children ranges between 0.42 - 0.97 ml,
while in adults it ranges between 0.63 - 1.46 ml.
1. Type B curve with normal ear canal
volume suggests otitis media.
2. Type B curve with small canal
volume suggests that the ear canal could be
occluded by the presence of wax, or the probe of
the impedance audiometer has not been properly
placed.
3. Type B curve with large canal
volume suggests that there could be perforation of
the ear drum. (so middle ear volume is added up to
volume of ear canal)
Type C - Peak compliance is significantly below zero,
indicating negative pressure (sub-atmospheric) in the middle
ear space. This finding is often indicative eustachian tube
dysfunction. compliance peak is less than -150 daPa
note:
Testing for the presence of absence of perilymph
fistula:
Testing implies presence or absence of a fistula (ie,
an abnormal opening in the inner ear labyrinthine system).
This can be indirectly assessed by the presence of
intense giddiness along with nystagmus when the external
canal pressure in increased by increasing the probe pressure.
This sign is also known as the Hennebert's sign. This sign is
manifested only in the presence of perilymph fistula.
Testing function of eustachian tube:
A negative or positive air pressure is created (-200 to
+200) in middle ear & person is asked to swallow 5 times in 20
sec.
the ability to equate the pressure indicates normal
tubal function.
also used to test patency of grommet placed in
tympanic membrane in cases of serous otitis media.
Physical volume of ear canal:(Equivalent ear
canal volume)Includes the volume between
probe tip(of impedence audiometer) & tympanic
membrane ,if tympanic membrane is intact. (or)
volume of ear canal & middle ear space if
tympanic membrane is perforated.
method:
it is derived from acoustic admittance of volume of
air medial to probe.
Under reference conditions ,a given volume of air
has a known acoustic admittance, which can be used to
calculate the equivalent volume of air.
eg;when a 226Hz probe tone is used ,1cubic cm of air
has admittance of 1acoustic mmho under standard atmospheric
conditions.
so, if admittance of air b/w probe & tympanic
membrane is 1.5 acoustic mmho ,then equivalent volume is 1.5
cubic cm.
Average ear canal volumes for children are 0.42-0.97
mL. Average adult volumes are 0.63-1.46 mL.
Used in case of Type B tympanograms(flat) to know the
cause:
Type B curve with normal ear canal volume
suggests otitis media.
Type B curve with
small canal volume
suggests that the ear
canal could be occluded
by the presence of wax,
or the probe of the
impedance audiometer
has not been properly
placed.
Type B curve with large canal volume suggests
that there could be perforation of the ear drum. (so middle
ear volume is added up to volume of ear canal)
Static compliance:
Measure of middle ear mobility.
it is measures in equivalent volume in cc's, based on
2 volume measurements.
C1= made with tympanic membrane in position of
poor compliance with +200 mmH2O in external canal.
C2= made with tympanic membrane at max
compliance
C1-C2= Static compliance, which cancels out the
compliance due to column of air in external canal.the
remainder is compliance due to middle ear
mechanisms.
static compliance is low when value is less than
0.28cc & high when more than 2.5cc
its major contribution is to differentiate b/w fixed
middle ear & middle ear discontinuity.
Acoustic reflex:
principle:
Contraction of the stapedius muscle occurs with loud
sounds, producing a measurable change in compliance.
When the stapedius muscle contracts in response to
a loud sound, that contraction changes the middle ear
immittance. This change in immittance can be detected as a
deflection in the recording.
A significant change in middle ear immittance
immediately after the stimulus is considered an acoustic reflex.
A stapedial muscle contraction in response to an
intense signal occurs bilaterally in normal ears with either
unilateral or bilateral stimulation. This reaction occurs because
the stapedial reflex pathway has both ipsilateral and
contralateral projections
we should first know a few basic terminology used
here,
the immittance change caused by stapedius
muscle contraction is measured in the ear containing the probe
tip --this is PROBE EAR.
the ear receiving the stimulus to activate the
reflex is --STIMULUS EAR.
either ear can be stimulus ear --i.e, the
stimulus can even originate from the probe tip as well as
from the ear phone on the opposite ear.
Absent reflex means even 125dB of sound
doesn't elicit contralateral reflex.
Elevated reflex : the patient's threshold is
compared to respective 90th percentiles that apply to his
hearing threshold for the frequencies tested.If ART falls above
the 90th percentile it is considered elevated.
Ipsilateral or uncrossed acoustic reflex: here
the stimulus is presented to the probe ear ,which is the same
ear in which immitance change is measured.
Contralateral or crossed acoustic reflex: here
the reflex is measured in the ear with probe tip ,but stimulus is
given to opposite ear.
"right contralateral
acoustic reflex" means stimulus
is in right ear & probe in left ear.
"left contralateral acoustic reflex" means
stimulus is in left ear & probe in right ear.
Probe ear principle:acoustic reflexes are usually
absent when there is conductive pathology in probe ear.
stimulus ear principle: a conductive disorder in
the stimulus ear reduces the stimulus level reaching the
cochlea by the amount of air-bone gap,As a result ART is
elevated by the amount of air-bone gap.
2 basic acoustic reflex tests:
1. acoustic reflex threshold(ART)--lowest stimulus
which produces reflex
2. acoustic reflex decay--measure of how long reflex
lasts when stimulus is kept for a period of time
ART:
Acoustic reflex thresholds generally are determined in
response to stimuli of 500, 1000, 2000, and 4000 Hz. For
screening purposes, or for a general check of the pathway's
integrity, usually test at 1000 Hz.
Range of ART :loud sound 70-100dB above
threshold of hearing of that particular ear is used
The greater the hearing loss, the higher the
acoustic reflex threshold for conductive hearing loss.
For sensorineural hearing loss, acoustic reflex
thresholds may be within the normal range, particularly for
mild-to-moderate hearing losses with recruitment.
Absent(means even 125dB of sound doesn't elicit
contralateral reflex) in:
1. Reflexes usually are absent or cannot be recorded
if the patient has type B tympanograms; therefore,
acoustic reflexes generally are not tested in these
ears.
For example,
if the ear canal is occluded with cerumen, a type B
tympanogram with low volume will be recorded. In this case,
acoustic reflexes cannot be measured because middle ear
immittance is not being measured. (Cerumen blocks the signal.)
For a type B tympanogram with normal volume (as in
otitis media) no pressure peak for immittance is obtained. The
pressure between the ear canal and middle ear are not
equilibrated, and acoustic reflexes cannot be recorded.
For a type B tympanogram with high volume (as in the
presence of patent pressure equalization tubes or perforated
tympanic membranes), an open exchange of air occurs
between the ear canal and middle ear; thus, any contraction of
the stapedius muscle cannot be measured.
2. In the presence of severe-to-profound sensorineural
hearing loss in the stimulated ear, acoustic reflexes may
be absent secondary to insufficient stimulation.
3. Similarly, a conductive component may attenuate
the signal intensity, preventing sufficient stimulation in
the stimulated ear or precluding a clear response in the
recording ear.
Typical patterns for the interpretation of acoustic
reflex abnormalities are as
follows:
With unilateral conductive
deafness:
In the given image ,right contralateral reflex
absent or elevated (due to probe principle)
but left ear ipsilateral absent (due to both probe
ear & stimulus ear principles)
So in ipsilateral both principles are operative
(ie,stimulus is attenuated due to conductive disorder
& even immitance change cannot be measured due to
conductive disorder)
Due to this double effect ipsilateral acoustic reflex are so
sensitive to conductive disorder.
Bilateral conductive disorder:
both contra & ipsilateral
reflexes absent in both ears.
Acoustic reflex decay test (adaptation):
it is measure of how long the response lasts if
stimulus is kept on for a period of time.
The acoustic reflex decay test is used to assess
the integrity of CN VIII. Using a stimulus of either 500 or 1000
Hz, a contralateral continuous tone is presented for 10
seconds at a stimulus level 10 dB above the acoustic reflex
threshold for that stimulus frequency in that ear. This
suprathreshold acoustic reflex then is recorded over the 10-
second stimulation period. If the amplitude of the recorded
deflection on the screen decreases by 50% or more within 10
seconds, the test is considered positive.
In some cases of muscular or neuromuscular
disorder, tone decay results also may be positive secondary to
muscle fatigue.
along with ART (absent or elevated),acoustic
reflex decay is used to detect retro-cochlear disorder
useful in:
test hearing in infants & young children - since it is
objective test
find malingers- a person who feigns total deafness &
doesn't give any response on PTA ,but shows +ve stapedial
reflex is malingerer.
detect cochlear pathology - presence of stapedial
reflex at lower intensities like 40 -60 dB than usual 70dB
indicates recruitment & thus cochlear type hearing loss
lesions of facial nerve- absence of reflex when
hearing is normal indicates lesion of facial nerve proximal to
nerve to stapedius.
lesion of VIIIth nerve- Acoustic reflex decay test
lesion in brain stem:ipsilateral reflex present but
contralateral absent indicates lesion is in crossed pathways in
brain.
Bekesy audiometry:A subject-controlled auditory threshold testing
procedure.method:
the patient has to press a button
the button controls a motor ,which in turn controls
attenuator & a pen.
patient is told to hold button down when he can hear a
tone & release it when he cannot.
holding the button causes the intensity to fall &
releasing to rise.intensity change 2.5dB/sec
this course of events will cause the level of of tone to
rise & fall around the patient's threshold.
the pen tracks the level of of tone on a paper resulting
in zigzag pattern around the patient's threshold.
the width of the zigzags is EXCURSION WIDTH &
the patient's threshold is midpoint of these thresholds.
Conventional Bekesy audiometry:
Bekesy audiograms are obtained either
one frequency at a time(fixed frequency Bekesy
audiometry):one frequency for a given period of time.
test frequency changes from low to high.(sweep
frequency Bekesy audiometry):test tone increases smoothly
from 100 to 10,000Hz at rate of 1 octave/sec
Each Bekesy audiogram is obtained twice
once with a continuous tone (CONTINUOUS
TRACING)
other with a tone that pulses on & off 2.5
times/sec(PULSED TRACING)
Types of tracing:the shifting of tracing is result of tone decay.
continuous tracing -(C)
pulsed tracing-(P)
types pattern seen in illustration
I (C) &(P) normal hearing &
tracings conductive hearing
overlap loss.
II continuous cochlear loss
& pulsed
tracings
overlap up
to 1000Hz
& then (C)
tracing
falls by an
amount
that is less
than 20dB
&
excursions
of (C)
decreases
up to even
3-5dB
wide
III (C) tracing retrocochlear lesion
falls below
(P) at 100
to 500Hz
,even up to
40-50dB
separation
& even up
to
audiometer
limits
IV (C) falls retrocochlear lesion
below (P)
at
frequencies
up to
1000Hz by
more than
25dB but
not to
audiometer
limits.
V (C) above non -organic hearing
(P) loss(pseudohypoacusis)
Modifications of Bekesy audiometry:
Reverse Bekesy tracings:
Here the sweep of the (C) tracing is from high to
low frequency & compared with (P) tracing from low to
high.
advantage is that the patients with sensorineural
loss as a result of retrocochlear pathological condition in
the region of brain stem had more hearing loss with this
procedure.
Bekesy comfort loudness test:
Patients are instructed to respond to
suprathreshold stimuli rather than threshold stimuli.(at a
comfortable loudness rather than too loud or too soft)
reason:retrocochlear disorders initially appear at
suprathreshold levels.
http://www.medicalmcq4all.com/book/ent4.php
http://www.medicalmcq4all.com/book/ent4.php
http://www.medicalmcq4all.com/book/ent4.php